Woher wissen wir, dass Quantenverschränkung unabhängig von der Entfernung funktioniert?

Es wird gesagt, dass die Quantenverschränkung unabhängig von der Entfernung funktioniert. 2 Teilchen können verschränkt werden und Informationen werden augenblicklich geteilt, selbst wenn sie Lichtjahre voneinander entfernt sind.

Aber woher wissen wir, dass das bei einem so großen Abstand zwischen den beiden Teilchen noch funktioniert? Ich kann mir Experimente in einem Labor oder sogar auf gegenüberliegenden Seiten des Planeten vorstellen, aber nicht mit Lichtjahren Abstand zwischen ihnen. Woher wissen wir das?

Siehe auch : physical.stackexchange.com/q/3158/2451 und darin enthaltene Links.
„Informationen werden sofort geteilt“ Das wird von vielen QM-Interpretationen rundweg abgelehnt und von den meisten zumindest als irreführend angesehen.
Ich habe einen hoch bewerteten Kommentar entfernt, der eine Antwort hätte sein sollen, und ungefähr sechs Antworten darauf. Bitte verwenden Sie Kommentare nur , um Verbesserungen für den Beitrag vorzuschlagen, an den sie angehängt sind.

Antworten (10)

Abstand ist keine relativistische Invariante. Sei A das Ereignis, das dadurch gekennzeichnet ist, dass ich mit dem Schreiben dieser Antwort begonnen habe, und B das Ereignis, bei dem ich mit dem Schreiben aufhöre. Im Bezugssystem meines Schreibtisches ist der Abstand zwischen A und B Null, aber im Bezugssystem eines sich bewegenden Beobachters 0,9999999 c relativ zur Erde beträgt die Entfernung zwischen A und B Millionen von Kilometern.

Wenn die Trennung zwischen zwei Ereignissen raumartig ist, dann können wir nur sagen, dass sie in anderen Frames auch raumartig getrennt sind. Der Abstand zwischen ihnen kann beliebig groß gemacht werden, indem ein geeigneter Referenzrahmen ausgewählt wird.

Alles, was wir also brauchen, ist ein Experiment, das zeigt, dass die Verschränkung über raumähnliche Intervalle funktioniert, und daraus folgt, dass sie über beliebig große Entfernungen funktioniert. Ein solches Experiment ist Guerreiro et al., http://arxiv.org/abs/1204.1712 .

In Ihrem ersten Beispiel ist die Trennung zwischen den Ereignissen zeitlich. Und Sie argumentieren, dass der Abstand zwischen ihnen beliebig groß gemacht werden kann, indem Sie sie von einem geeigneten Referenzrahmen aus beobachten. Das Teil klingt richtig. Aber wie folgt daraus, dass dies auch für Ereignisse mit raumartiger Trennung gilt? Wird die Längenkontraktion nicht dazu führen, dass es umgekehrt ist, sodass die raumartige Trennung auf einen beliebig kleinen Abstand gebracht werden kann, aber nicht größer?
Zeit=Raum=Raumzeit. Das erste Beispiel ist raumartig, um es leichter verständlich zu machen. Der Abstand zwischen A und B ist nicht immer Null. Es ist Null zum Referenzrahmen auf der Erde, weil er sich mit der gleichen Geschwindigkeit wie der Schreibtisch bewegt und daher die Bewegung der Erde um die Sonne oder der Sonne um die Milchstraße nicht als Faktor in seinen Gleichungen aufzeichnen kann. Die Bewegung des Planeten würde die Entfernung von AB im Laufe der Zeit relativ ungleich Null machen. Somit ist der Raum für dieselbe Gleichung nicht in Bezug auf eine absolute Nullbewegung definiert, sondern eine relative Bewegung zum Beobachter , der sich auch auf der Erde befindet.
Warte was? Das scheint OP überhaupt nicht zu beantworten, fügen Sie seiner Frage einfach mehr Details hinzu. Guerreiro zeigt eine statistisch signifikante Verteilung in jeder Entfernung, in der er gemessen hat; OP fragt im Wesentlichen: "Wie können wir wissen, dass die Verteilung mit der Entfernung unveränderlich ist?"
@kasperd: Die zeitähnlichen und raumähnlichen Fälle sind nicht gleich. Ich habe den zeitähnlichen Fall nur zur Veranschaulichung der Tatsache besprochen, dass Entfernungen nicht unveränderlich sind. Dass der Abstand beliebig groß gemacht werden kann, folgt für den raumartigen Fall aus der Niederschrift der Lorentz-Transformation. Wenn die Ereignisse in einem Rahmen simultan sind, dann ist ihre Trennung in einem anderen Rahmen um einen Faktor größer γ , die beliebig groß gemacht werden kann.
@fectin: Der Abstand kann beliebig groß gemacht werden, einfach durch Ändern des Bezugsrahmens.
Natürlich kann es. Und wie wirkt sich das auf die Verteilung aus?
Ich argumentiere im Geiste vieler anderer hier geposteter Antworten, dass es bei dieser Frage im Wesentlichen um das empirische Testen von Modellen geht. So gesehen ist diese Antwort nicht ausreichend. Diese Antwort besagt, dass, wenn Sie den Vorhersagen der speziellen Relativitätstheorie und der Quantenmechanik bis zu sehr hohen Geschwindigkeiten glauben, aktuelle Experimente bereits mit der Quantenverschränkung übereinstimmen, die über beliebig große Entfernungen "arbeitet". Aber der Geist dieser Frage zweifelt an den mathematischen Theorien, bis empirische Beweise geliefert werden ...
Es gab noch keine empirischen Beweise dafür, dass die Quantenmechanik und die spezielle Relativitätstheorie bei diesen großen Geschwindigkeits- und Entfernungsskalen wie erwartet funktionieren sollten, daher denke ich nicht, dass diese Antwort als Antwort auf die Frage des OP ausreicht.
@jgerber: Es gab noch keine empirischen Beweise dafür, dass die Quantenmechanik und die spezielle Relativitätstheorie bei diesen großen Geschwindigkeits- und Entfernungsskalen wie erwartet funktionieren sollten . An welche Geschwindigkeiten und Skalen denken Sie? Die Lorentz-Invarianz wurde mit unglaublich hoher Präzision bei vielen verschiedenen Geschwindigkeiten und Skalen getestet. Ein Beispiel bei Geschwindigkeiten sehr nahe bei c wäre das CERN-FTL-Neutrino-Debakel, das später gelöst und als in Übereinstimmung mit der Lorentz-Invarianz befunden wurde.
Das ist ein fairer Punkt. Es ist mir nicht klar, dass die Teilchenexperimente auf der Art der Verschränkung beruhen, die in Bell-Testexperimenten gemessen wird, und daher ist es möglicherweise nicht fair, die Verschränkungstheorie auf der Grundlage dieser anderen Experimente auf hohe Geschwindigkeiten zu extrapolieren, obwohl ich meinen Standpunkt einräume ist schwach. Mein Hauptpunkt ist, dass Ihr Beitrag eine theoretische Extrapolation erfordert, die gerechtfertigt sein kann oder nicht, je nachdem, wie pedantisch der Leser sein mag. Oder besser gesagt, je nachdem, wie sehr der Leser aktuelle Beweise mit dem Ziel zusammenfassen möchte, die derzeit akzeptierten Theorien zu erweitern.
(1/2) @jgerber: Wir wissen aus dem Experiment, dass sich die Verschränkung viel schneller als mit Lichtgeschwindigkeit bewegen müsste: arxiv.org/abs/1303.0614 Und wir wissen aus der Theorie, dass sich die Verschränkung mit jeder endlichen Geschwindigkeit schneller bewegt als c könnte für superluminale Kommunikation verwendet werden: nature.com/articles/nphys2460
(2/2) Ich glaube nicht, dass es im Prinzip möglich ist zu sagen, dass es keine denkbare Theorie gibt, die eine solche superluminale Signalisierung zulassen könnte, während sie mit bestehenden Experimenten und den grundlegenden Vorstellungen von Kausalität kompatibel ist, die für die physische Welt erforderlich sind haben keine kohärente Logik dazu. Es ist nie möglich, definitiv zu sagen, dass die Theorie, die man hat, die einzig richtige Theorie ist. Aber es ist fair zu sagen, dass, da eine Theorie der Ausbreitung von Verschränkung eine vollständige Überarbeitung der Grundlagen der modernen Physik erfordern würde, die Beweislast sicherlich bei der Person liegt, die behauptet, dass sie lebensfähig sein kann.
@Rococo "Aber es ist fair zu sagen, dass die Beweislast sicherlich bei der Person liegt, die behauptet, dass sie lebensfähig sein kann, da eine Theorie der Ausbreitung der Verschränkung eine vollständige Überarbeitung der Grundlagen der modernen Physik erfordern würde." Ich stimme dieser Aussage zu. Aber ich füge die Warnung hinzu, dass wir Fragen wie die OPs nicht abschreiben sollten, weil sie "ein Umschreiben der modernen Physik erfordern". Man kann moderne Bell-Tests als Präzisionstests der Gesetze der speziellen Mechanik und der Relativitätstheorie interpretieren. Präzisionstests können, sobald sie extrem genug werden, neue Physik aufdecken.
Vielleicht werden wir feststellen, dass wir, egal wie sehr wir uns auch bemühen, möglicherweise keine Bell-Korrelationen zwischen einem erdgebundenen Teilchen und einem Teilchen auf dem Mond erkennen können. Dies bedeutet, dass entweder mit der Relativitätstheorie oder der Quantenmechanik etwas nicht stimmt. Vielleicht hilft das Ausspionieren dieser Inkonsistenz beispielsweise dabei, Theorien zur Quantengravitation zu finden. Die Vorsicht ist also, dass wir weiterhin Fragen wie die OPs stellen sollten, oder wir könnten fälschlicherweise eine Theorie extrapolieren und etwas übersehen.

Der Begriff „Quantenverschränkung existiert“ ist eine Teilmenge des Begriffs „es existiert eine einzige quantenmechanische Wellenfunktion, die das System beschreibt“.

Wenn Sie die Wellenfunktion kennen, dh eine mathematische Beschreibung davon haben, hat die Theorie der Quantenmechanik Einschränkungen für Quantenzahlen und ihre Erhaltung, also sind Quantenzahlen "verschränkt", weil es eine einzige Wellenfunktion gibt, die das System beschreibt, wie ayc auch sagt.

Nehmen wir ein einfaches Beispiel, den Pi0-Zerfall. Es zerfällt in zwei Photonen/Gammas. Der pi0 hat Spin Null. Aus Erhaltungen der Quantenzahlen (in diesem Fall Drehimpulserhaltung) in der Zerfallswellenfunktion muss eines der Photonen Spin +1 und das andere -1 haben, wobei die Richtung die Richtung ihrer Bewegung ist.

Sobald der Zerfall stattgefunden hat und es keine Wechselwirkungen gibt, sind die Spins für den Experimentator unbekannt, aber sie werden durch die Wellenfunktion definiert.

Wenn ich einen im Labor messe und der andere Alpha Centauri erreiche, kenne ich seinen Spin, indem ich den einen im Labor gemessen habe und die Drehimpulserhaltung verwendet habe.

Das ist deine Verwirrung:

und Informationen werden sofort geteilt, selbst wenn sie Lichtjahre voneinander entfernt sind

Die Information wird nicht von den Photonen geteilt, die Information liegt im Kopf des Experimentators, der im Labor sitzt.

Woher wissen wir, dass das wahr ist? Weil wir die quantenmechanische Theorie mit unzähligen Experimenten validiert haben, der Mathematik vertrauen und alle Informationen hier im Labor extrapolieren.

Bei der Verschränkung geht es um mehr als nur eine gewöhnliche Wellenfunktion, die das System beschreibt; Der Punkt ist, dass der Zustand nicht als Tensorprodukt über Hilbert-Räume für die Subsysteme zerlegt werden kann. Wenn ja, dann gibt es keine Verstrickung. Zum Beispiel, ( | ↑↓ + | ↓↑ ) / 2 ist verstrickt, aber | ↑↓ ist nicht.
@JG Ich habe ein experimentelles Beispiel gegeben. Die Quantenzahlerhaltung im beobachtbaren Ergebnis gilt unabhängig von der mathematischen Formulierung der Wellenfunktion. "Verschränkung" beschreibt eine zusätzliche Einschränkung unter Verwendung von Erhaltungsgesetzen. Wie Wellenfunktionen Erhaltungssätze auferlegt werden, ist ein mathematisches Problem.
Der letzte Absatz dieser Antwort hätte ungefähr zweihundert Jahre nach den Principia über die Newtonsche Mechanik gesagt werden können.
@ Martin Argerami . Sicher, nur Messungen führen uns an der Nase herum zur Quantenmechanik. Wenn neue, genauere (oder was auch immer) Messungen die Quantenmechanik ungültig machen, wird sie statt der zugrunde liegenden Schicht aller klassischen Modelle zu einer der Schichten mit ihrem Gültigkeitsbereich. Es ist "Schildkröten ganz unten".
Aber es gibt immer noch den "gruseligen" Teil, wenn Sie das Experiment so konzipieren, dass die Beobachtungsmethode bestimmt, in welche Richtung der Spin beobachtet wird. Was kann die Wellenfunktion bis zu dieser Beobachtung sagen, außer dass das andere Teilchen das Gegenteil davon hat eine unbekannte Drehung?
@Michael Die Wellenfunktion ist Mathematik, also enthält sie nur Erhaltungsgesetze. Dasselbe gilt für mechanische Trajektorien, die das Auftreffen auf den Boden vorhersagen. Die Wellenfunktion sagt nur Wahrscheinlichkeiten voraus. Das ist der Unterschied. die Wahrscheinlichkeitsvorhersagen, nicht Trajektorien und Massenverteilungen. Es ist nicht die Beobachtungsmethode, jedes korrekte Experiment sollte die gleiche Wahrscheinlichkeitsverteilung ergeben, sonst ist das Modell ungültig.

Die kurze Antwort ist "Nein". Tatsächlich ist die grundlegende Tatsache des Lebens in der Wissenschaft, dass wir sozusagen nie wissen: Wir haben eine Theorie, sie macht Vorhersagen, die wir testen, und wenn der Test fehlschlägt, dann wissen wir mit Sicherheit, dass die Vorhersage falsch war, grob gesagt. Wenn es nicht scheitert, wissen wir eigentlich nicht, dass wir Recht hatten – aber wir können uns berechtigt fühlen, zu glauben, dass wir es sind. Also genau genommen, auch wenn wir das erlebt haben 10 7 Mal, wenn ein Stein herunterfällt, wissen wir immer noch nicht, ob er es beim nächsten Mal tun wird.

Natürlich würden einige – die meisten – sagen, dass dies unnötige Sophistik ist, und ich würde in den meisten Fällen dazu neigen, innerhalb vernünftiger Grenzen zuzustimmen. Wir haben in Experimenten bestätigt, dass die Verschränkung über Entfernungen von mehreren hundert Kilometern zu funktionieren scheint, glaube ich, aber zu behaupten, dass sie über Lichtjahre funktionieren wird, ist meiner Meinung nach immer noch sehr viel Spekulation oder Wunschdenken.

===BEARBEITEN===

Angesichts der bisherigen Kommentare denke ich, dass ich vielleicht meine Position klarstellen sollte. Die grundlegende Einsicht, die in dem, was wir die wissenschaftliche Methode nennen, formalisiert ist, ist, dass wir niemals durch Experimente absolute Gewissheit über die Gültigkeit der Theorie erlangen können – die einzige absolute Wahrheit kommt in Form von Falsifikation; Eine erfolgreiche Theorie ist eine, bei der es uns konsequent nicht gelungen ist, die Vorhersagen zu falsifizieren. Das müssen wir immer im Hinterkopf behalten, wenn wir auf der Grundlage unserer Theorie extrapolieren – denken Sie nur an Newtons Gravitationstheorie: Sie funktioniert wunderbar für (fast) alle praktischen Zwecke, wir verwenden sie, wenn wir unsere kieselgroßen Satelliten um mehrere herumschleudern Planeten und schaffen es immer noch, das Ziel meistens zu treffen - aber es konnte die Präzession des Perihels von Merkur nicht vollständig erklären.

Eines Tages – hoffentlich bald – werden wir Situationen finden, in denen sowohl GR als auch QM versagen. Dann machen wir den nächsten Schritt nach vorne.

===BEARBEITEN2===

Ich sehe, dass meine Antwort abgelehnt wurde - das ist in Ordnung für mich, aber wenn Sie so denken, fügen Sie bitte einen Kommentar hinzu, damit ich erfahren kann, ob ich einen Fehler gemacht habe, unhöflich war oder etwas anderes. Ich werde es dir nicht übel nehmen.

Ich war bis zum letzten Satz bei dir. Es ist weder. Es ist eine Folge der Theorie, die es tut. Wir haben festgestellt, dass die Theorie ihren Wert hat. Es ist so wahr wie die Verschränkung bei genau 2,050421 cm. Ungetestet, aber wir gehen davon aus, bis wir einen Grund finden, die Theorie zu verbessern.
Ich würde diese Antwort 100 Mal positiv bewerten, wenn ich könnte. Ich würde zustimmen, dass die Extrapolation von mehreren hundert Kilometern bis zu Lichtjahren (oder Milliarden von Lichtjahren) eine signifikante Extrapolation über die Bedingungen hinaus ist, unter denen die Theorie experimentell validiert wurde, die unser Vertrauen in das noch gültige Ergebnis erheblich verringern sollte . Das bedeutet nicht, dass die Theorie bei größeren Entfernungen falsch ist, nur dass wir es wirklich nicht wissen, da wir bei solchen Entfernungen keine Beweise dafür oder dagegen haben. Und intellektuelle Demut verlangt von uns, dies anzuerkennen.
Ich freue mich, Ihnen mitteilen zu können, dass es Milliarden von Menschen (mich eingeschlossen) gibt, die wissen, dass Steine ​​herunterfallen. Es tut mir leid, dass Sie irgendwann von der Realität abgelenkt wurden und eine axiomatische Definition von Wissen akzeptiert haben. Es ist nur in Mathematik und Religion nützlich, nicht aber in den Naturwissenschaften, da ihnen Axiome fehlen.
Ich denke, j4nd3r53n hat tatsächlich darauf hingewiesen, dass wir wissen, dass Steine ​​​​herunterfallen, aber das gilt für einen Fall, in dem die Wahrscheinlichkeit so nahe bei 1 liegt, dass sie vernünftigerweise an 1 angenähert werden kann. In dem von der Frage betroffenen Fall können wir das nicht einmal Schätzen Sie die Wahrscheinlichkeit für große Entfernungen ab, da wir noch nie Experimente über diese Entfernungen durchgeführt haben (im Gegensatz zu dem Felsexperiment, das wir unglaublich oft durchgeführt haben). Das Beste, was wir tun können, ist, die Wahrscheinlichkeit für kürzere Entfernungen, für die es Experimente gibt, abzuschätzen und dann auf die längeren Entfernungen zu extrapolieren.
"So nah an 1, dass es ungefähr 1 entspricht" Ich meine, dass wir (meines Wissens nach) noch nie beobachtet haben, dass ein Stein nicht herunterfällt. So ziemlich jeder würde das „wissen, dass der Stein herunterfallen wird“ nennen, mich eingeschlossen. Aber technisch gesehen ist es das nicht, zumindest nicht, wenn man sich nicht der Philosophie des Empirismus anschließt.
Die Antwort wäre "wir nicht" und nicht "nein", da die Frage lautet "woher wissen wir das?".
„Wir alle wissen, dass Steine ​​herunterfallen“, bis wir einen unbeschleunigten Rahmen entdecken.

Es wird gesagt, dass die Quantenverschränkung unabhängig von der Entfernung funktioniert. 2 Teilchen können verschränkt werden und Informationen werden augenblicklich geteilt, selbst wenn sie Lichtjahre voneinander entfernt sind.

Um es ganz klar zu sagen: Verschränkung kann nicht zur Kommunikation genutzt werden , egal wie lang oder wie kurz die räumliche Trennung ist. Dies wird in diesem Thread ausführlicher erklärt , aber das Grundprinzip ist folgendes:

  • Angenommen, Sie haben zwei verschränkte Teilchen, sagen wir, Spins im Auf-Ab-Verschränkungszustand | + | .
  • Nehmen Sie weiter an, dass Sie das erste Teilchen entlang der Auf-Ab-Basis messen, und Sie erhalten das Ergebnis s . Dann projiziert das auch den zweiten Spin auf den Zustand s .
  • Darüber hinaus ist es je nach Ihrer Interpretation von QM und Ihrer allgemeinen Haltung zu seinen grundlegenden Problemen möglich, dies als sofortige Aktion auf dem zweiten Qubit zu interpretieren, unabhängig von der Entfernung zwischen ihnen.
  • Allerdings: Sie haben keine Kontrolle darüber, ob Sie das Ergebnis erhalten s = oder s = , also haben Sie keine Kontrolle über die "Nachricht", die gesendet wird.

Dies kann im No-Communication-Theorem weiter formalisiert werden , das im Grunde besagt, dass, wenn das System den Regeln der Quantenmechanik gehorcht, keine Spielereien verwendet werden können, die Sie möglicherweise entlang dieser Linien ziehen könnten, um schneller als Licht zu kommunizieren. Die Quantenmechanik ist eine vollständig kausale Theorie, dh es gibt keine Szenarien, in denen die Auswirkungen einer bestimmten Ursache außerhalb ihres zukünftigen Lichtkegels beobachtet werden können.

Andererseits ist es möglich , die in den Aufzählungspunkten oben beschriebene Situation so zu interpretieren, dass die Partikel auf nicht kausale Weise schneller als Licht miteinander „kommunizieren“ und sich dann „verschwören“, um diesen FTL-Kommunikationskanal nicht verfügbar zu machen zu jedem makroskopischen Experiment ─ aber die zweite Hälfte dieser Kombination ist entscheidend und darf niemals ausgelassen werden. Einige Leute sind damit einverstanden, aber ich finde es als philosophische Position zutiefst unbefriedigend.

Um es ganz klar zu sagen, das oben beschriebene Protokoll beruht nicht wirklich auf Verschränkung, und es ist anfällig für eine "Bertlmanns Socken" -Erklärung, wo Sie gerade eine setzen und ein Spins in unbeschrifteten Kartons und versenden Sie sie. Diese Art der Erklärung lokaler versteckter Variablen reicht jedoch nicht aus, um den gesamten Satz von Messergebnissen zu erklären, die unter Verwendung verschränkter Zustände möglich sind: Theorien versteckter Variablen sind durch Bells Theorem eingeschränkt , um eine Reihe von Ungleichungen in Bezug auf die Arten und Mengen von Korrelationen zu erfüllen, die sie zeigen können , und es gibt mehrere Experimente , die zeigen, dass quantenmechanische Systeme diese Ungleichungen routinemäßig brechen.


Allerdings mit all diesen Präzisionen:

Es wird gesagt, dass die Quantenverschränkung unabhängig von der Entfernung funktioniert.

Ja, das ist richtig: Soweit wir wissen, funktioniert das alles unabhängig von der räumlichen Trennung zwischen den Partikeln.

Aber woher wissen wir, dass das bei einem so großen Abstand zwischen den beiden Teilchen noch funktioniert? Ich kann mir Experimente in einem Labor oder sogar auf gegenüberliegenden Seiten des Planeten vorstellen, aber nicht mit Lichtjahren Abstand zwischen ihnen. Woher wissen wir das?

Wir haben keine Beweise dafür außer der Tatsache, dass die Theorie funktioniert hat, um jedes Experiment zu erklären, um das wir sie gebeten haben, auf allen Skalen, die wir geschafft haben, Arbeitstests der Theorie zu konstruieren, und die Tatsache, dass alle unsere astronomischen Beobachtungen der Physik von Orten, die für uns unzugänglich sind (vom Vorhandensein von Helium in der Sonne bis zum thermischen Spektrum des kosmischen Mikrowellenhintergrunds) können mit denselben physikalischen Gesetzen erklärt werden, die wir mit erdgebundenen Labors testen.

Es ist durchaus möglich, dass ein beliebiges physikalisches Gesetz außerhalb des Bereichs bricht, in dem wir es getestet haben, weshalb wir es immer wieder in neueren und größeren Regimen testen; in der Tat, wenn wir solche Abweichungen finden, wären sie viel interessanter als die Feststellung, dass es überall gleich ist. Solange wir jedoch kein solches Ergebnis finden, gibt es keine Beweise dafür, dass dies nicht der Fall ist.

Im Laufe der Zeit gelang es den Experimentatoren, die Entfernung zu erhöhen, über die die Verschränkung nachgewiesen werden konnte, bis zu vielen Kilometern.

Lassen Sie mich einen Vergleich mit der Geschichte des Elektromagnetismus anstellen:

Als Maxwell die Theorie des Elektromagnetismus veröffentlichte, die wir heute als Maxwellsche Gleichungen kennen, war eine der Implikationen der Theorie, dass es möglich sein sollte, eine sich ausbreitende elektromagnetische Welle zu erzeugen und diese elektromagnetische Welle zu empfangen. Heinrich Hertz wollte das überprüfen. Der von ihm konstruierte Emitter wurde durch Überspringen eines Funkens auf einen sehr kurzen, sehr starken Wechselstrom gestoßen. Der starke Wechselstrom verursachte eine elektromagnetische Welle. Der Empfänger befand sich ziemlich genau neben dem Sender. Der Empfänger nahm die elektromagnetische Welle auf.

Natürlich war die Entfernung, die diese erste empfangene Welle zurückgelegt hatte, sehr überwältigend, nur ein Meter oder so. Aber die Sache war die: Nichts in der Theorie deutete darauf hin, wie weit sich elektromagnetische Wellen ausbreiten können. Tatsächlich wurde bald entdeckt, dass es eine Reihe von Frequenzen gibt, die von der Ionosphäre reflektiert werden, und so gab es innerhalb kürzester Zeit nach der Hertz-Demonstration Sendeantennen, die ein Signal aussendeten, das auf der ganzen Welt empfangen werden konnte.

Ähnliches gilt für die Quantenverschränkung. Die ersten Vorführungen erfolgten über relativ kurze Distanzen, die Abmessungen eines Einzelbankaufbaus. Aber für Elementarteilchen sind sogar Entfernungen von Zentimetern riesig. Also: Sobald die Quantenverschränkung auf einem Einzelbankaufbau demonstriert wurde, war das Bild ziemlich klar: Nichts in der Theorie deutet auf eine Grenzentfernung hin, und eine Demonstration über eine Entfernung von Zentimetern ist bereits eine Demonstration über eine große Entfernung.

Je länger die Entfernung, desto geringer ist natürlich der Ertrag. Je länger die Transitzeit, desto größer ist die Wahrscheinlichkeit, dass die Verschränkung unterwegs verloren geht. Um also eine akzeptable Anzahl von Bestätigungen pro Sekunde zu haben, müssen Sie größere Populationen verschränkter Teilchen erzeugen und senden. Aber wir haben allen Grund zu der Annahme, dass es keine Entfernung gibt, jenseits derer der Ertrag auf Null geht.

Interessanterweise hat Maxwell alle Gleichungen in einer einzigen Quaternion-Form (Art 623, IIRC) zusammengefasst, die im Wesentlichen die 4D-Raum-Zeit-Kontraktion ist. Es hängt davon ab, ob man glaubt, dass die Metrik zuerst kommt oder der 4d-Raum.

Die Antwort lautet: „Wir wissen es nie.“ Ob die Gesetze der Physik auf der anderen Seite des Jupiters genauso gelten, lässt sich nicht mit Sicherheit sagen. Natürlich haben alle Beobachtungen, die wir jenseits von Jupiter gemacht haben, die Schlussfolgerung gestützt, dass die Gesetze der Physik überall gleich sind, aber wir werden es nie mit Sicherheit wissen.

Dementsprechend müssen wir eigentlich nur "empirisch ausreichende" Beweise dafür haben, dass die Effekte über große Entfernungen wirken. Aus diesem Grund stellen wir fest, dass Entfernungen wirklich nicht der größte einschränkende Faktor zu sein scheinen. Die interessantere Grenze ist die Lichtgeschwindigkeit. Wenn die Verschränkung dies gut modelliert, dann ist es eine nützliche Theorie.

Geschwindigkeit ist einfacher als Distanz. Wenn ich zwei Messungen im Abstand von nur Nanosekunden zeitlich festlegen kann, kann mein Experiment in einem Raum stattfinden und dennoch verschränkungsähnliche Effekte aufweisen, die nicht durch Übertragungen erklärt werden können, die sich mit Lichtgeschwindigkeit ausbreiten. Wenn die Messung durch ein gemeinsames Signal auf halbem Weg zwischen den Teilen meines Experiments ausgelöst wird, können wir trivialerweise sehen, dass ich mit einem verschränkten Paar auf eine Weise interagieren kann, die viel schneller ist, als es die Lichtgeschwindigkeitskommunikation erlauben würde, weil keine Zeit für eine Runde bleibt Reisen.

Ist das also ein Beweis dafür, dass die Effekte über eine Entfernung von Lichtjahren wirken? Nein. Wenn ich es in einem 10-Fuß-Raum gemacht habe, beweist es nichts darüber, ob Verschränkung in einem 11-Fuß-Raum funktioniert. Aber alle unsere bisherigen Experimente haben gezeigt, dass die Entfernung überhaupt keinen Einfluss hat.

Leider erinnere ich mich nicht an alle Details, aber es sind die Experimente, die als Reaktion auf das EPR-Paradoxon durchgeführt wurden, die uns zwangen, die Nichtlokalität eines verschränkten Zustands zu akzeptieren.

Diese Experimente erzeugen im Grunde einige Paare verschränkter Teilchen, hauptsächlich Photonen, senden sie in einiger Entfernung voneinander und messen beide auf zufällige Weise. Es kann gezeigt werden, dass die genaue Statistik dieser Messungen voraussetzt, dass eine Messung die andere tatsächlich beeinflusst. Dieses Ergebnis ist das, was die Quantenmechanik vorhergesagt hat und was Einstein, Podolsky und Rosen absurd fanden (weshalb es als Paradoxon bezeichnet wird). Afaik, die Konsequenz dieser Ergebnisse ist, dass wir entweder die Nicht-Lokalität verschränkter Zustände akzeptieren oder unsere Vorstellung von physikalischer Realität hinterfragen müssen.

Diese Experimente wurden mit verschiedenen Entfernungen durchgeführt, ich weiß nicht, was die aktuelle Rekordentfernung ist. Aber es ist im Grunde sinnlos, über Entfernungen zu streiten, wenn die beiden Messungen so weit voneinander entfernt sind, dass das Licht den Ort der anderen Messung nicht erreichen kann, bevor diese Messung tatsächlich stattfindet. Das ist für Photonen einfach zu erreichen: Wenn die verschränkten Photonen in entgegengesetzte Richtungen gehen und Sie eine Messstation nur einen Meter entfernt platzieren, finden die Messungen im Bezugssystem des Labors zum gleichen Zeitpunkt statt, aber zu zweit Meter auseinander. Das reicht aus, um eine Kommunikation mit Lichtgeschwindigkeit zwischen den verschränkten Teilchen auszuschließen.

Schlagen Sie zum Beispiel nach Ghost Imaging, das Zwillingsphotonen verwendet, um Objekte zu erkennen, die von einem Photon gesehen werden, indem der Effekt auf dem anderen beobachtet wird. en.wikipedia.org/wiki/Ghost_imaging Der Großteil des Problems besteht darin, zu entscheiden, ob wir eine 4D-Raumzeit haben (alle Punkte einzigartig) oder ob die 4D-Messung nur eine Projektion aus einer anderen Dimensionalität ist (z. B. alle Punkte auf dem Lichtkegel sind gleich, wenn innerhalb einer sehr kleinen Ursprungsentfernung - der Verschränkungsentfernung).

Die Prämisse, dass „Quantenverschränkung überhaupt funktioniert “, ist falsch – es ist keine Handlung erforderlich – ebenso wie die, dass „Informationen geteilt werden“ (unabhängig davon, ob sie augenblicklich sind oder nicht). Quantenverschränkung ist in keiner Weise ein Vektor für die Übertragung von Informationen, und sie ist wirklich nicht tiefer , als zwei Bälle zu nehmen, einen roten und einen blauen, jeden in eine Schachtel zu legen, ohne sie anzusehen, die Schachteln zu trennen, eine Schachtel zu öffnen und zu sehen eine rote Kugel, und schlussfolgern, dass sich in der anderen Box eine blaue Kugel befindet.

Als ich die obige Analogie ursprünglich als Kommentar gepostet habe, wurde ein Einwand erhoben, dass sie irreführend ist und impliziert, dass die Quantenverschränkung mit (lokalen) verborgenen Variablen erklärbar ist. Das ist natürlich nach Bells Theorem sowie vielen experimentellen Ergebnissen unmöglich , und die „nicht tiefer als“-Analogie sollte nicht dahingehend interpretiert werden, dass es eine versteckte Variable gibt. Wenn Sie wirklich wollen, können Sie sich vorstellen, es als Gedankenexperiment im Stil einer Schrödinger-Katze aufzustellen und die Bälle in einem (annähernd) isolierten Teil des Systems gemäß einem echten Zufallsprozess zu verteilen, der außerhalb nicht beobachtet wird. Aber das ist nebensächlich.

Der Punkt ist, dass es, wie in der Ballanalogie, bei der Quantenverschränkung darum geht, Schlussfolgerungen aus einer Beziehung zu ziehen, bei der, bedingt durch die vorherige Beobachtung des Aufbaus , eine Korrelation zwischen bestimmten Observablen und dem Ergebnis besteht.

Dies alles ist die Folge eines allgemeinen Prinzips, dass es in QM wirklich nichts Tieferes gibt als bedingte Wahrscheinlichkeiten in einem riesigen Modell, das bedeutungslos wäre, um irgendwelche Vorhersagen zu treffen, ohne auf Beobachtungen zu stützen.

Ich betrachte die Frage, inwieweit die Quantenverschränkung so funktioniert:

Solange die Verschränkung als mysteriöses Quantenphänomen betrachtet und in Begriffen von Wellenfunktionen oder anderen ähnlichen mathematischen Beschreibungen diskutiert wird, ist es schwer, sich vorzustellen, dass es etwas ist, das große Entfernungen über Raum (und Zeit) überleben kann.

Wenn Sie jedoch direkt zum Ursprung zurückgehen – wir haben die Verschränkung entdeckt, als wir mit korrelierten Eigenschaften experimentierten. Eigenschaften, von denen wir wissen, dass sie korreliert sind, weil - wenn sie es nicht wären - Erhaltungsgrößen - wie der Drehimpuls - nicht erhalten bleiben würden. Während GR einige Lücken hinterlässt und das Universum Energie erzeugen kann, gibt es im Universum keine Hinweise darauf, dass die Erhaltung streng lokal ist und Energie und Impuls und andere konservierte Eigenschaften trivialerweise gebrochen werden können, wenn Sie nur ein bisschen mehr Platz hinzufügen Mitte des Experiments.

dh jede Entfernung, die wir zeigen können, ist zu kurz, um eine Symmetrieverletzung zu verursachen, ist auch eine Entfernung, von der wir erwarten können, dass die Quantenverschränkung über sie hinweg wirkt.

Ich persönlich möchte sagen, dass die Aussage bisher nicht genau oder zumindest nicht eindeutig ist. Wenn wir im Allgemeinen glauben, dass die Struktur der Raumzeit eng mit der Verschränkung zusammenhängt, dann sollten wir sehr vorsichtig sein, in einem Fall, in dem es um Verschränkung geht, über „Entfernung/Zeit/Geschwindigkeit“ zu sprechen.

Meine Meinung ist, dass, wenn wir mit zwei verschränkten Teilchen spielen, ER=EPR zu behaupten scheint, dass die „normale“ Raumzeitstruktur durch die Verschränkung gebrochen (oder zumindest gestört) wird. In dieser Situation ist es nicht sehr streng zu sagen, „die beiden Teilchen sind weit entfernt“, da das kleine Wurmloch zwischen ihnen nur eine Abkürzung darstellen könnte, so dass ihr „Abstand“ 0 ist. Tatsächlich zeigt die beobachtete Tatsache, dass verschränkte Teilchen einen Momentan haben Korrelation scheint zu behaupten, dass sie derselbe Punkt SIND.

Schließlich glaube ich, bevor wir die Quantengravitation vervollständigen können, um QM und GR zu integrieren, sollte es keine eindeutige Antwort auf diese Frage geben.